7
$\begingroup$

Let $A$ be an abelian variety over an algebraically closed field $k$.

Let $\phi:A\to A$ be an étale isogeny (over $k$). Suppose that the set $\cup_{r\geq 0}({\rm ker}\,\phi^{\circ r})(k)$ is Zariski dense in $A$. Here $\phi^{\circ r}:=\phi\circ\dots\circ\phi$ ($r$-times).

Now let $\lambda:A\to A$ be an endomorphism and suppose that for all $r\geq 0$, we have $$\lambda(({\rm ker}\,\phi^{\circ r})(k))\subseteq ({\rm ker}\,\phi^{\circ r})(k)\,\,\,\,(*).$$

Does it follow that we have $$\phi^{-1}\circ\lambda\circ \phi=\lambda\circ c$$ in ${\rm End}(A)_{\bf Q}$ for some $c\in{\rm End}(A)_{\bf Q}$ which commutes with $\phi$? Or more generally, what relation in ${\rm End}(A)_{\bf Q}$ between $\lambda$ and $\phi$ does $(*)$ imply?

One may of course ask the same question for any isogeny (not just étale) and formulate a more general condition involving finite group schemes but the case above is the case I am interested in.

$\endgroup$
4
  • 4
    $\begingroup$ Couldn't you have something like $\lambda\circ\phi=- \phi\circ\lambda$ (or, more generally, automorphisms of finite order)? $\endgroup$
    – ACL
    Mar 29, 2017 at 11:47
  • $\begingroup$ @ACL: thank you. You are quite right (silly me!). In fact one could replace $-1$ by any endomorphism commuting with $\phi$. I shall modify the question accordingly but I am bit confused about what to expect. $\endgroup$ Mar 29, 2017 at 12:49
  • 1
    $\begingroup$ No, not any endomorphism : equality of degrees forces automorphism. $\endgroup$
    – ACL
    Mar 29, 2017 at 17:00
  • 1
    $\begingroup$ @ACL: only if $\lambda$ is an isogeny. $\endgroup$ Mar 30, 2017 at 8:42

1 Answer 1

3
$\begingroup$

Remark. The condition that $\bigcup_{r \geq 0} \ker(\phi^r)(k)$ is Zariski dense is not needed. Indeed, if it is not satisfied, replace $\phi$ by $[\ell] \circ \phi$ for $\ell$ invertible in $k$. Since $([\ell] \circ \phi)^r = [\ell^r] \circ \phi^r$, the kernel contains all $\ell^r$-torsion, which is Zariski dense as $r \to \infty$.

Moreover, $[\ell] \circ \phi$ satisfies $(*)$ if $\phi$ does. Indeed, if $x \in \ker([\ell^r] \circ \phi^r)$, then $$([\ell^r] \circ \phi^r) (\lambda(x)) = \phi^r(\lambda([\ell^r]x)) = 0,$$ since $[\ell^r]x \in \ker \phi^r$ and by condition $(*)$ for $\phi$. Thus, $\lambda(x) \in \ker([\ell^r] \circ \phi^r)$, which proves condition $(*)$ for $[\ell] \circ \phi$.

Finally, $\phi^{-1} \circ \lambda \circ \phi$ is unaffected by this replacement, since scalars are central.

Negative answer to your question. Let $A = E \times E$ be the square of an elliptic curve. Let $\phi \colon (x,y) \mapsto (y,x)$ be the swap isomorphism, and let $\lambda \colon (x,y) \mapsto (x,0)$ be the projection. Condition $(*)$ is now trivially satisfied (there is no kernel).

But $\phi^{-1} \circ \lambda \circ \phi$ is now the other coordinate projection $(x,y) \mapsto (0,y)$. This is not of the form $\lambda \circ c$ for any $c$, for example since the images don't agree (even after inverting scalars). Nor is it of the form $c \circ \lambda$, because the kernels don't agree.

Is there anything we can say? The best I can do is the following: condition $(*)$ implies, by the fundamental theorem on homomorphisms¹, that there exist maps $\lambda_r \colon A \to A$ such that the diagrams $$\begin{array}{ccc}A & \stackrel\lambda\longrightarrow & A \\ {\tiny{\phi^r}}\downarrow\ \ & & \ \ \downarrow\tiny{\phi^r} \\ A & \stackrel{\lambda_r}\longrightarrow & A \end{array}$$ commute. In fact, this is equivalent to condition $(*)$: the converse follows from the diagram.

But as we saw above, the relation between $\lambda_1$ and $\lambda$ is not something simple like $\lambda_1 = \lambda \circ c$ or $\lambda_1 = c \circ \lambda$ for some $c$.


¹One should say some words about set-theoretic inclusion and scheme-theoretic inclusion in $(*)$. This should be fine because $\ker(\phi^r)$ is an étale $k$-scheme, and $k$ is algebraically closed.

$\endgroup$
1
  • $\begingroup$ Right! Thank you very much. That clarifies it - one cannot say anything more than what the diagram expresses in general. $\endgroup$ Mar 30, 2017 at 8:39

Your Answer

By clicking “Post Your Answer”, you agree to our terms of service and acknowledge you have read our privacy policy.

Not the answer you're looking for? Browse other questions tagged or ask your own question.